Difference between revisions of "Mock AIME 1 2010 Problems/Problem 3"
Pommukutti (talk | contribs) (Created page with "Lo wassup homies") |
m (page was empty, so i added a see also section, statement of the problem, and statement of the answer) |
||
Line 1: | Line 1: | ||
− | + | == Problem == | |
+ | Let <math>AB</math> be a line segment of length <math>20 \sqrt{2}</math>, and let <math>S</math> be the set of all points <math>P</math> such that <math>m \angle APB \geq 45^{\circ}</math>. Find the last three digits of the largest integer less than the area of <math>S</math>. | ||
+ | |||
+ | == Solution == | ||
+ | <math>\boxed{284}</math>. | ||
+ | |||
+ | == See Also == | ||
+ | *[[Mock AIME 1 2010 Problems]] | ||
+ | *[[Mock AIME 1 2010 Problems/Problem 2|Followed by Problem 2]] | ||
+ | *[[Mock AIME 1 2010 Problems/Problem 4|Followed by Problem 4]] |
Revision as of 06:51, 2 August 2024
Problem
Let be a line segment of length , and let be the set of all points such that . Find the last three digits of the largest integer less than the area of .
Solution
.